2018 February CA Bar Forum

Discussions related to the bar exam are found in this forum
Forum rules
Anonymous Posting

Anonymous posting is only appropriate when you are sharing sensitive information about bar exam prep. You may anonymously respond on topic to these threads. Unacceptable uses include: harassing another user, joking around, testing the feature, or other things that are more appropriate in the lounge.

Failure to follow these rules will get you outed, warned, or banned."
jduckits

New
Posts: 17
Joined: Sun Jul 02, 2017 8:40 pm

Re: 2018 February CA Bar

Post by jduckits » Fri Mar 02, 2018 8:14 pm

Baller31 wrote:
jduckits wrote:
Baller31 wrote:
jduckits wrote:Did anyone actually make any attempt to raise defenses to nuisance? I couldn’t remember the rule statements for substantial and unreasonable interference. I made them up... Not sure how many points I will get...
Yes. The defense of laches, because the plaintiff was annoyed about the smokehouse smell for quite some time but did not bring a suit for equitable relief seeking an injunction or possible monetary relief within a reasonable time. It's a weak defense to nuisance but a well established on point defense according to case law. Didn't see much of anything else on point though.

That was a good defense to raise. Quick thinking!
Thanks. Really had to think fast for that one. I figured that it was maybe smart to devote more time to the condemnation/takings action since they gave us that issue, making it likely they expected more analysis on it, and just use prior knowledge from past cases on the private nuisance action coupled with possible defenses. Apparently both private nuisance and condemnation is a real big issue in California.
Yeah I felt like the takings one wasn’t that bad though. You didn’t have to argue regulatory vs. actual taking, which I think would have been a deeper analysis. My main analysis was under Public Purpose, Just Compensation, & inverse condemnation

Call2222222

New
Posts: 11
Joined: Wed Nov 22, 2017 6:54 pm

Re: 2018 February CA Bar

Post by Call2222222 » Fri Mar 02, 2018 8:34 pm

Wow that topic section for the essays is brutal. I’m glad i was able to pass July 2017 because it seems this was a way harder exam. With those subjects, i don’t think i would of passed. I wish the best of luck to everyone who took this exam. Don’t beat yourself up too much, this was a difficult one and a hard one to grade!

sophophone

New
Posts: 2
Joined: Fri Mar 02, 2018 8:28 pm

Re: 2018 February CA Bar

Post by sophophone » Fri Mar 02, 2018 8:36 pm

I added that the attorney breached the duty of competence because it was not necessary to put the expert on in the first place, since the roof company had advised the seller that the roof leaked. My thought was why was the expert needed. I saw that as a breach of competence, as well as a breach of financial responsibility since he was wasting the client's money by paying an expert witness. There weren't any facts re money spent, so I may be off. I also talked about breach of loyalty because he put someone on the stand who he knew would testify adversely to his client.
Mxmasterr wrote:
Bla Bla Bla Blah wrote:
cgc210 wrote:Can anyone please remind me what the first part of the question with Prof responsibility was about? I cannot remember for the life of me
The PR question was tied into the property-contracts question, re: the quitclaim deed where there was a question of whether seller misrepresented the potentially faulty roof, or had a duty to disclose and chose to omit. It asked us to analyze under the ABA and the CA rules.

I analyzed the duty of loyalty, which is essentially to represent the clients interests, and not attorney's own. The duty of competence/care, which only differs in CA because it adds to the ABA rule that attorney is to have the requisite mental, and physical, ability to competently represent. Arguably, putting an expert on the stand when the lawyer was fully aware that the expert had come to an entirely different opinion in another matter on the same issue is shows lack of competence, and certainly is not in the client's best interest.

I also analyzed the ABA rule on lawyer testimony (i.e., not at all, in this situation at least). In CA, it is allowed, but only if there is agreement among counsel and the judge permits.

That was pretty much all there was to it. I was honestly expecting more, considering that the CA Bar questions and model answers from past administrations of the test seemed to have pretty hard core PR questions that required much more analysis. For instance,
July 2017 Question 2 would have made me have an aneurysm if something like it would have popped up on this test... that one was a all out nuanced PR topic, not just a sidenote issue like ours was.
No I think the question turned on false testimony and whether the lawyer knew the expert was lying. The lawyer did his own research that contradicted the expert ... It screamed is this guy perjuring himself. The lawyer then repeated the claim to the jury which could be a false statement of fact and violate duty of candor and fairness by putting on and arguing false evidence. Then he offered an opinion from his own investigation u can't do that. He told the jury something he did that was not in evidence from personal knowledge that's not allowed. Also the lawyer being a witness stuff. I thought that whole question was testing lawyer knowledge of perjury though.

rayforoc

New
Posts: 50
Joined: Fri May 12, 2017 9:36 pm

Re: 2018 February CA Bar

Post by rayforoc » Fri Mar 02, 2018 10:38 pm

Long read if you're feeling it. Sincerest apolygizes for all the cussing but I wanted to write how I am actually feeling.

So I am taking it again. TAKING IT AGAIN (Al Pacino from Scent of a Woman Voice).

From what I can recall. This is my wrap up and I will be brutally honest about my prep too.

Prep:
- I did 1100 adaptibar questions ending at 77%;
- went through S&T;
- did all four tests on NBCE website during final week (85%, 90%, 90%, and 88%);
- bought NCBE newest study guide and only got through the first ten civ pro questions;
- never read one outline because I though I remembered shit from July, and realize now how fucking careless and stupid that was.

Essays:
- Outlined (and I mean like almost wrote them out) 10 Torts, 10 Contracts, 8 Evidence + 2 CA Evidence, 10 Property, 9 Con Law, 10 BA + 2 Partnerships and 2 Agency, 5 Trusts, and 6 Wills, 10 Crimes... I didnt touch any civ pro, comm prop, pr, or remedies.

PT:
- did not do any prep here other than to browse through some notes from July.

I didnt outline pr and remedies because I ran out of time and should have prioritized those but didnt because I thought that doing pr and remedies within the BA and Contracts essays I was outlining would be sufficient. This is stupid. Also, not practicing any fucking PTs was stupid, and not fucking timing myself, at all, was stupid (see below to find out why). Like this is the one fucking area where we cannot be arrogant when prepping. I am fucking arrogant sometimes and arrogant bc of so many reasons no of which are important and all of which are stupid, and I think its like actually an infection that is fucking my life up slowly (here because only an idiot would not read outlines, completely skip PT prep, oh and not touch pr and remedies--the two guaranteed subjects... fucking go punish yourself). Arrogance is terrible. Its nasty. It hurts people, and it hurts yourself in my ways than you can imagine. and I am trying to get better each day at this side of me. I Looked through all the comments thus far and you ladies and gents are beasts (you should all be proud), and I am too, its just that I am kind of reckless and I am afraid it really fucked me up this time. The first time I took this I scored something like 1292, and then 1377. I hope I passed but yeah. Hope. The operative word. Hope. A noun. To want something to happen. Use it in a sentence. I unfortunately hope that a certain unqualified and worthless Congresswoman wins a certain midterm election in my district because I would hate to have to run against someone in my own party in 2020 and utterly fuck their lives up in a primary battle.

The Exam

MBEs: Nothing major and felt good both times although I missed a couple easy ones because I forgot the BLL, and the afternoon session was a bit shit, so I am assuming I got one of the easier AM sets seeing as how mostly everyone thought their AM was hard and afternoon easy. Outlines, bud, should have read your fucking outlines. Should have gone through all the questions in the study aid too. Like that's just nonsense to fuck that up. Btw, NCBE needs to write better civ pro questions.

Essays:

1. Hit formation re contract analysis (prob a wait and might actually piss the grader off?), one sentence on terms, then dove into misrepresention and non-disclosure. Then hit PR. From what I rememeber it was Duty of Care, Duty of Loyalty (specifically, conflict of interest and I said lawyer's intersest in saving money with shady expert v. client's interests of winning the case, something like that and worded better), Duty of Dignity and Dicorum, Duty of Competence, Duty to Communicate, and maybe one more? My BLL was absolutely not spot on on some of them (Hence the mistake of not study PR omg).

2. Hit Standing, Organizational Standing (quickly dismissed it tho I think, right? yeah I dont think this worked), then Taxpayer standing (bc dude can get it on this if his/her cause of action is related to one of the religious clauses, right?, then ripeness, then hit constitutional claims and started with the requirement of state action, then went into the religions first, specifically the Establishment Clause first but forget to talk about sect/no sect preference, and did do Lemon here, I think I fucked it up and did Lemon for the Free Exercise Clause, then hit P&I of the 14th but did not hit Due Process, then hit EP and did classification as something like prisoners who are allowed to use/adhere to their recognized religions v. the dude, and since it was religion I thought that was a fundamental right and applied strict scrutiny, also, I distinctly remember doing separate analysis for the book and for the tea on several of the issues.

3. Hit Public Nuisance, then private nuisance exception under public, then hit Private Nuisance, then Trespass (and didnt call it Trespass To Land But Rather just Trespass), then Tresspass To Chattels (wire fence), then hit Negligence cuz 10 years of this shit is fucking negligent (went through quick analysis with both cardozo and andrews standard of care included). Do it matter which way you landed on for all three nuisance claims? I forgot which way I went. I feel like instantly it does matter. Didnt do conversion, IIED, Neg IIED, but did do a super, suuuuper shit eminent domain and Takings analysis. Super shitty like the graders going to see eminent domain think "yeah this dude should not be practicing," and then my takings was jusst shit because I was down to literally seconds at that point. Btw, happy that I at least got through both MBE sections (last time I didnt).

4. Hit search and seizure (govt conduct, S&S, standing? idk forgot already), then went into warrant requirement if all are present, then said warrant was fine after going through several different analyses, then since Officer Eager raced to the house I hit warrantless exceptions, and of them go her in except to Exigent Circumstances??????????????? That's what i called it. Not Hot pursuit, but Exigent Fucking Circumstances. This is literally, if I remember correctly, how I did it too "Warrantless Exception #5 = Exigent Circumstances." Yeah, so fucked that up. But then Eager got in. So next part of the essay was the stuff right, so Bomb is fine bc girl could be hiding in closet, analysis, but other two were not, and did then exclusionary rule analyses for all three. Then, I completely fucked the rule for Kidnapping up and even remember mentioning aggravated kidnapping but no because not for money, but then maybe MAYBE turned it around a bit under Attempted Kidnapping analysis writ large. I thought about writing about how other two could come in if used for a grand jury indictment, but i dont remember if this is correct so I opted not to fuck essay 4 up even more.

5. Wills was a fucking mess. full stop. First, I started by talking about the overall Community Property rule, then right below it I had a little section titled “CA Rule Application” talking about the three areas ways its valid (state, domiciled, death). Next, I talked about validity of 2001 will (present testamentary intent, capacity, and will formalities), but prob did not get the BLL precise for each of those. Then I hit revocation I think (revival, DRR, physical act, subsequent will, codicil). Then codicil, and I think somewhere it the abovementioned analysis I mentioned the CA rule regarding the signature. I think I then went into devises under the will. Start with Nell and hit putative spouse, dismissed, then co-habitants, dismissed, then intestate share, but didn’t get that right, then lapse and anti-lapse, dismissed. Then with Ann I talked about her gift being specific not general because it was for a sum certain from SPECIFICALLY his SP? So definitely fucked that up because it was a general devise… hmmm… that sucks. I also hit Ann on anti-lapse and gave her all that shit from his first will. Bob got his gift and something else, forgot, and Carol got intestate share because he was an omitted child through adoption… CA recognizes adoption as synonymous with blood child, and presumes omission to be unintentional absent evidence to the contrary. No existed here plus intention to give because his other two children were gifted 20k I think. START employers under class gift, can be left open, and probably fucked that rule up too.

PT: this is why I think I will be retaking this again for the fourth time. This is terrible and fucking heartbreaking because this thing has cost me so much over the last almost two years now. It’s also cost others. Anyway. So I had a little over an hour to do the PT and quickly banged through and created my outline: I. Contract Analysis, II. Gift Analysis (a. unconditional, b. conditional), III. Standing Analysis. But then it all went to shit. Didn’t have time. I go all the rules down for each section, but by that time was left with only ten minutes analysis with our facts. So I hit one paragraph on how its not a contract, and maybe four sentences on how it’s an unconditional gift. Finally, I was down to two minutes and literally wrote one sentence dismissing P’s complaint for lack of claim for which relief could be granted given that this was not a contract, so this PT will be a 50, maybe even a 45 or 40.

Out of five essays I think I might have gotten this: 70, 65, 60, 55, 50… maybe those are even less, and a 45 or 50 on the PT.

I am going to get back in the gym, figure out the job situation, sort out my material, get my shit together and next week I will start by reading the Barbri big books I have, like the huge outlines. I am also going to fucking memorize all my subject outlines to the point where I can write shit down from memory. I am doing 15 essays for each subject. Each fucking one. And not just out line them. I am going to do it to where I have these facts patterns memorized and answers via the answers outlined below. I have to think about what I am going to do regarding MBE prep.

This bar is passable because I fucking felt it this time around. But then I felt it slip out on my hands, again, and its just heartbreaking. Some of us are ok and other aren’t. Some of us have lost a little and others have lost a lot. I have lost love. I have lost almost everything. I have nothing right now. But that’s ok because I will push back and keep going, and with the right corrections I will pass. Its just that I am getting sick of this shit. I just turned 35. I have really nothing to show for it. I went into the navy after high school, then undergrad and grad at two great schools, ok, but fuck, this is getting ridiculous. Since the Navy I really have not had much work experience. Highly talented but just dragged down by this shit bar. I should have practice PTs under timed conditions. I should have done essays under timed conditions. I should have known the BLL better. I should have a better and quicker understand substantive law period.

/rant

Thank you for taking the time to read this if you made it this far. Sincerely, I mean that.

Update: I knowing I am freaking and very well might have passed. But then I maybe didn't. I think I passed the MBEs because I honestly felt pretty good both sessions, but if that PT and shit wills essays tank my written portion that will suck but will be a good personal lesson. Gym time.

Yogagirl

New
Posts: 14
Joined: Thu Jul 27, 2017 1:16 pm

Re: 2018 February CA Bar

Post by Yogagirl » Sat Mar 03, 2018 3:55 am

To long read, thank you. You are not alone. I did close to 2000 practice questions and the NCBE practice tests, but only scored an average of 68 percent so you are in great shape. Regarding the PT, I did the same thing in July and got a 50. They don't generally go below this score, so hope this helps to relieve your anxiety about it. I took a PT course in law school and completed seven prior to this administration (including rewriting the Davis PT from July) but it all went out the window for me, too. This was my third attempt, but intend to see this through. I scored 1351 in July and knew I had a steep mountain to climb. Hope is what continues to propel me upward.

As I read other comments realize that I missed some important issues (writing about the First Amendment with regard to the darn signs was a complete blunder, but once I fell down that rabbit hole had to finish what I started), but was also spot on for others. I am a old single mother of a child with a moderate disability and also a full time teacher; it took me five years to finish law school. I will do whatever it takes to cross the finish line and know that I am almost there.

I believe that many of us have given up more than we anticipated. As for me, I have absolutely no regrets. This is not for the weak.

Stay strong. Persevere.

Want to continue reading?

Register now to search topics and post comments!

Absolutely FREE!


User avatar
catechumen

New
Posts: 50
Joined: Wed May 21, 2014 11:20 pm

Re: 2018 February CA Bar

Post by catechumen » Sat Mar 03, 2018 4:55 am

Just wanted to say your impression of the exam is spot on. I felt the same way. Since you are former military too, I think you understand better than most that the closest thing to Bar stress is a drill sergeant yelling in your face or a battle drill with arty sims going off. I take that back, the Bar is worse. Much worse! I will take a DS over one of those stone cold heartless graders. At least DS's wanted us to succeed.

Civ Pro questions were garbage. I spent more time studying Civ Pro then other subjects by far, yet was unable to figure out what black letter law they were testing on half the time. Essays/PT were tough and hard to finish on time. Worse still the PT was confusing. Finished it, but it was short, and my analysis was short. I really think they wrote this test knowing that you could not write a passing score on every essay every time, there just was not enough time. You could write two good essays, and one trash essay.

For your sake and mine, I hope we passed. I have taken this awful test a bunch too and want to be done. Missed passing by 1 MBE last time... Yet as hard last time was, these dirt bags proceed to write a harder test. In fact most of the practicing attorneys fail. Minimum competence my tail!

Perhaps some day the SCOTUS will stop state bars from hazing us like this, but until then we just got to take it like a man. This is straight out of Animal House! "Assume the position... Thank you Sir may I have another!" https://www.youtube.com/watch?v=-7870l6ECBw

Yet we keep going back... And I have to ask why.

Mxmasterr

New
Posts: 42
Joined: Thu Mar 01, 2018 10:14 am

Re: 2018 February CA Bar

Post by Mxmasterr » Sat Mar 03, 2018 6:22 am

rayforoc wrote:Long read if you're feeling it. Sincerest apolygizes for all the cussing but I wanted to write how I am actually feeling.

So I am taking it again. TAKING IT AGAIN (Al Pacino from Scent of a Woman Voice).

From what I can recall. This is my wrap up and I will be brutally honest about my prep too.

Prep:
- I did 1100 adaptibar questions ending at 77%;
- went through S&T;
- did all four tests on NBCE website during final week (85%, 90%, 90%, and 88%);
- bought NCBE newest study guide and only got through the first ten civ pro questions;
- never read one outline because I though I remembered shit from July, and realize now how fucking careless and stupid that was.

Essays:
- Outlined (and I mean like almost wrote them out) 10 Torts, 10 Contracts, 8 Evidence + 2 CA Evidence, 10 Property, 9 Con Law, 10 BA + 2 Partnerships and 2 Agency, 5 Trusts, and 6 Wills, 10 Crimes... I didnt touch any civ pro, comm prop, pr, or remedies.

PT:
- did not do any prep here other than to browse through some notes from July.

I didnt outline pr and remedies because I ran out of time and should have prioritized those but didnt because I thought that doing pr and remedies within the BA and Contracts essays I was outlining would be sufficient. This is stupid. Also, not practicing any fucking PTs was stupid, and not fucking timing myself, at all, was stupid (see below to find out why). Like this is the one fucking area where we cannot be arrogant when prepping. I am fucking arrogant sometimes and arrogant bc of so many reasons no of which are important and all of which are stupid, and I think its like actually an infection that is fucking my life up slowly (here because only an idiot would not read outlines, completely skip PT prep, oh and not touch pr and remedies--the two guaranteed subjects... fucking go punish yourself). Arrogance is terrible. Its nasty. It hurts people, and it hurts yourself in my ways than you can imagine. and I am trying to get better each day at this side of me. I Looked through all the comments thus far and you ladies and gents are beasts (you should all be proud), and I am too, its just that I am kind of reckless and I am afraid it really fucked me up this time. The first time I took this I scored something like 1292, and then 1377. I hope I passed but yeah. Hope. The operative word. Hope. A noun. To want something to happen. Use it in a sentence. I unfortunately hope that a certain unqualified and worthless Congresswoman wins a certain midterm election in my district because I would hate to have to run against someone in my own party in 2020 and utterly fuck their lives up in a primary battle.

The Exam

MBEs: Nothing major and felt good both times although I missed a couple easy ones because I forgot the BLL, and the afternoon session was a bit shit, so I am assuming I got one of the easier AM sets seeing as how mostly everyone thought their AM was hard and afternoon easy. Outlines, bud, should have read your fucking outlines. Should have gone through all the questions in the study aid too. Like that's just nonsense to fuck that up. Btw, NCBE needs to write better civ pro questions.

Essays:

1. Hit formation re contract analysis (prob a wait and might actually piss the grader off?), one sentence on terms, then dove into misrepresention and non-disclosure. Then hit PR. From what I rememeber it was Duty of Care, Duty of Loyalty (specifically, conflict of interest and I said lawyer's intersest in saving money with shady expert v. client's interests of winning the case, something like that and worded better), Duty of Dignity and Dicorum, Duty of Competence, Duty to Communicate, and maybe one more? My BLL was absolutely not spot on on some of them (Hence the mistake of not study PR omg).

2. Hit Standing, Organizational Standing (quickly dismissed it tho I think, right? yeah I dont think this worked), then Taxpayer standing (bc dude can get it on this if his/her cause of action is related to one of the religious clauses, right?, then ripeness, then hit constitutional claims and started with the requirement of state action, then went into the religions first, specifically the Establishment Clause first but forget to talk about sect/no sect preference, and did do Lemon here, I think I fucked it up and did Lemon for the Free Exercise Clause, then hit P&I of the 14th but did not hit Due Process, then hit EP and did classification as something like prisoners who are allowed to use/adhere to their recognized religions v. the dude, and since it was religion I thought that was a fundamental right and applied strict scrutiny, also, I distinctly remember doing separate analysis for the book and for the tea on several of the issues.

3. Hit Public Nuisance, then private nuisance exception under public, then hit Private Nuisance, then Trespass (and didnt call it Trespass To Land But Rather just Trespass), then Tresspass To Chattels (wire fence), then hit Negligence cuz 10 years of this shit is fucking negligent (went through quick analysis with both cardozo and andrews standard of care included). Do it matter which way you landed on for all three nuisance claims? I forgot which way I went. I feel like instantly it does matter. Didnt do conversion, IIED, Neg IIED, but did do a super, suuuuper shit eminent domain and Takings analysis. Super shitty like the graders going to see eminent domain think "yeah this dude should not be practicing," and then my takings was jusst shit because I was down to literally seconds at that point. Btw, happy that I at least got through both MBE sections (last time I didnt).

4. Hit search and seizure (govt conduct, S&S, standing? idk forgot already), then went into warrant requirement if all are present, then said warrant was fine after going through several different analyses, then since Officer Eager raced to the house I hit warrantless exceptions, and of them go her in except to Exigent Circumstances??????????????? That's what i called it. Not Hot pursuit, but Exigent Fucking Circumstances. This is literally, if I remember correctly, how I did it too "Warrantless Exception #5 = Exigent Circumstances." Yeah, so fucked that up. But then Eager got in. So next part of the essay was the stuff right, so Bomb is fine bc girl could be hiding in closet, analysis, but other two were not, and did then exclusionary rule analyses for all three. Then, I completely fucked the rule for Kidnapping up and even remember mentioning aggravated kidnapping but no because not for money, but then maybe MAYBE turned it around a bit under Attempted Kidnapping analysis writ large. I thought about writing about how other two could come in if used for a grand jury indictment, but i dont remember if this is correct so I opted not to fuck essay 4 up even more.

5. Wills was a fucking mess. full stop. First, I started by talking about the overall Community Property rule, then right below it I had a little section titled “CA Rule Application” talking about the three areas ways its valid (state, domiciled, death). Next, I talked about validity of 2001 will (present testamentary intent, capacity, and will formalities), but prob did not get the BLL precise for each of those. Then I hit revocation I think (revival, DRR, physical act, subsequent will, codicil). Then codicil, and I think somewhere it the abovementioned analysis I mentioned the CA rule regarding the signature. I think I then went into devises under the will. Start with Nell and hit putative spouse, dismissed, then co-habitants, dismissed, then intestate share, but didn’t get that right, then lapse and anti-lapse, dismissed. Then with Ann I talked about her gift being specific not general because it was for a sum certain from SPECIFICALLY his SP? So definitely fucked that up because it was a general devise… hmmm… that sucks. I also hit Ann on anti-lapse and gave her all that shit from his first will. Bob got his gift and something else, forgot, and Carol got intestate share because he was an omitted child through adoption… CA recognizes adoption as synonymous with blood child, and presumes omission to be unintentional absent evidence to the contrary. No existed here plus intention to give because his other two children were gifted 20k I think. START employers under class gift, can be left open, and probably fucked that rule up too.

PT: this is why I think I will be retaking this again for the fourth time. This is terrible and fucking heartbreaking because this thing has cost me so much over the last almost two years now. It’s also cost others. Anyway. So I had a little over an hour to do the PT and quickly banged through and created my outline: I. Contract Analysis, II. Gift Analysis (a. unconditional, b. conditional), III. Standing Analysis. But then it all went to shit. Didn’t have time. I go all the rules down for each section, but by that time was left with only ten minutes analysis with our facts. So I hit one paragraph on how its not a contract, and maybe four sentences on how it’s an unconditional gift. Finally, I was down to two minutes and literally wrote one sentence dismissing P’s complaint for lack of claim for which relief could be granted given that this was not a contract, so this PT will be a 50, maybe even a 45 or 40.

Out of five essays I think I might have gotten this: 70, 65, 60, 55, 50… maybe those are even less, and a 45 or 50 on the PT.

I am going to get back in the gym, figure out the job situation, sort out my material, get my shit together and next week I will start by reading the Barbri big books I have, like the huge outlines. I am also going to fucking memorize all my subject outlines to the point where I can write shit down from memory. I am doing 15 essays for each subject. Each fucking one. And not just out line them. I am going to do it to where I have these facts patterns memorized and answers via the answers outlined below. I have to think about what I am going to do regarding MBE prep.

This bar is passable because I fucking felt it this time around. But then I felt it slip out on my hands, again, and its just heartbreaking. Some of us are ok and other aren’t. Some of us have lost a little and others have lost a lot. I have lost love. I have lost almost everything. I have nothing right now. But that’s ok because I will push back and keep going, and with the right corrections I will pass. Its just that I am getting sick of this shit. I just turned 35. I have really nothing to show for it. I went into the navy after high school, then undergrad and grad at two great schools, ok, but fuck, this is getting ridiculous. Since the Navy I really have not had much work experience. Highly talented but just dragged down by this shit bar. I should have practice PTs under timed conditions. I should have done essays under timed conditions. I should have known the BLL better. I should have a better and quicker understand substantive law period.

/rant

Thank you for taking the time to read this if you made it this far. Sincerely, I mean that.

Update: I knowing I am freaking and very well might have passed. But then I maybe didn't. I think I passed the MBEs because I honestly felt pretty good both sessions, but if that PT and shit wills essays tank my written portion that will suck but will be a good personal lesson. Gym time.
Exigent circumstances is correct, why do you think it was not? it's the overall category that hot pursuit and emergency medical care and dissapation and destruction of evidence fall under.

Sounds like u crushed everything but the pt(which does blow its worth 2 essays) and the wills one. The question said the will was valid ... So why analyze validity so much.

Got'eem

New
Posts: 15
Joined: Wed Feb 22, 2017 3:50 am

Re: 2018 February CA Bar

Post by Got'eem » Sat Mar 03, 2018 12:53 pm

Just remembered that I put “freedom of religion” and not free exercise clause as one of the issues. Put down the right elements but completely spaced out on the specific name of the issue

User avatar
a male human

Gold
Posts: 2233
Joined: Tue Mar 31, 2009 2:42 pm

Re: 2018 February CA Bar

Post by a male human » Sat Mar 03, 2018 3:39 pm

love4vinolaw wrote:
a male human wrote:Hello, hard-working citizens.

I'm looking for essay donations from the July 2017 exam because the free essay bank that I maintain is bare for that exam. In fact, it's the only administration I don't have any donations for yet.

Would any kind souls be willing to scan or take photos of their returned essays, for science? :) This is not for me; it's for your fellow students and future generations who want to stand on the shoulders of giants. Please PM me for where to send (or if you're a guest user of TLS, you may have to click around a bit to find my email), or I can reach out to you. Thanks in advance.
I can't do it before the CBX, but I will send you mine from the July 2017 next week when I get back. Do you want my PT also? I received a 75/80 on first and second review.
Hey love4vinolaw! Since I can't PM you anymore, just quoting you and hoping you'll see this as a reminder :) You can find my contact in this post http://www.top-law-schools.com/forums/v ... #p10298321

Thanks!

Want to continue reading?

Register for access!

Did I mention it was FREE ?


RezIpsy

New
Posts: 7
Joined: Fri Mar 02, 2018 7:06 pm

Re: 2018 February CA Bar

Post by RezIpsy » Sat Mar 03, 2018 10:29 pm

rayforoc wrote:Long read if you're feeling it. Sincerest apolygizes for all the cussing but I wanted to write how I am actually feeling.

So I am taking it again. TAKING IT AGAIN (Al Pacino from Scent of a Woman Voice).

From what I can recall. This is my wrap up and I will be brutally honest about my prep too.

Prep:
- I did 1100 adaptibar questions ending at 77%;
- went through S&T;
- did all four tests on NBCE website during final week (85%, 90%, 90%, and 88%);
- bought NCBE newest study guide and only got through the first ten civ pro questions;
- never read one outline because I though I remembered shit from July, and realize now how fucking careless and stupid that was.

Essays:
- Outlined (and I mean like almost wrote them out) 10 Torts, 10 Contracts, 8 Evidence + 2 CA Evidence, 10 Property, 9 Con Law, 10 BA + 2 Partnerships and 2 Agency, 5 Trusts, and 6 Wills, 10 Crimes... I didnt touch any civ pro, comm prop, pr, or remedies.

PT:
- did not do any prep here other than to browse through some notes from July.

I didnt outline pr and remedies because I ran out of time and should have prioritized those but didnt because I thought that doing pr and remedies within the BA and Contracts essays I was outlining would be sufficient. This is stupid. Also, not practicing any fucking PTs was stupid, and not fucking timing myself, at all, was stupid (see below to find out why). Like this is the one fucking area where we cannot be arrogant when prepping. I am fucking arrogant sometimes and arrogant bc of so many reasons no of which are important and all of which are stupid, and I think its like actually an infection that is fucking my life up slowly (here because only an idiot would not read outlines, completely skip PT prep, oh and not touch pr and remedies--the two guaranteed subjects... fucking go punish yourself). Arrogance is terrible. Its nasty. It hurts people, and it hurts yourself in my ways than you can imagine. and I am trying to get better each day at this side of me. I Looked through all the comments thus far and you ladies and gents are beasts (you should all be proud), and I am too, its just that I am kind of reckless and I am afraid it really fucked me up this time. The first time I took this I scored something like 1292, and then 1377. I hope I passed but yeah. Hope. The operative word. Hope. A noun. To want something to happen. Use it in a sentence. I unfortunately hope that a certain unqualified and worthless Congresswoman wins a certain midterm election in my district because I would hate to have to run against someone in my own party in 2020 and utterly fuck their lives up in a primary battle.

The Exam

MBEs: Nothing major and felt good both times although I missed a couple easy ones because I forgot the BLL, and the afternoon session was a bit shit, so I am assuming I got one of the easier AM sets seeing as how mostly everyone thought their AM was hard and afternoon easy. Outlines, bud, should have read your fucking outlines. Should have gone through all the questions in the study aid too. Like that's just nonsense to fuck that up. Btw, NCBE needs to write better civ pro questions.

Essays:

1. Hit formation re contract analysis (prob a wait and might actually piss the grader off?), one sentence on terms, then dove into misrepresention and non-disclosure. Then hit PR. From what I rememeber it was Duty of Care, Duty of Loyalty (specifically, conflict of interest and I said lawyer's intersest in saving money with shady expert v. client's interests of winning the case, something like that and worded better), Duty of Dignity and Dicorum, Duty of Competence, Duty to Communicate, and maybe one more? My BLL was absolutely not spot on on some of them (Hence the mistake of not study PR omg).

2. Hit Standing, Organizational Standing (quickly dismissed it tho I think, right? yeah I dont think this worked), then Taxpayer standing (bc dude can get it on this if his/her cause of action is related to one of the religious clauses, right?, then ripeness, then hit constitutional claims and started with the requirement of state action, then went into the religions first, specifically the Establishment Clause first but forget to talk about sect/no sect preference, and did do Lemon here, I think I fucked it up and did Lemon for the Free Exercise Clause, then hit P&I of the 14th but did not hit Due Process, then hit EP and did classification as something like prisoners who are allowed to use/adhere to their recognized religions v. the dude, and since it was religion I thought that was a fundamental right and applied strict scrutiny, also, I distinctly remember doing separate analysis for the book and for the tea on several of the issues.

3. Hit Public Nuisance, then private nuisance exception under public, then hit Private Nuisance, then Trespass (and didnt call it Trespass To Land But Rather just Trespass), then Tresspass To Chattels (wire fence), then hit Negligence cuz 10 years of this shit is fucking negligent (went through quick analysis with both cardozo and andrews standard of care included). Do it matter which way you landed on for all three nuisance claims? I forgot which way I went. I feel like instantly it does matter. Didnt do conversion, IIED, Neg IIED, but did do a super, suuuuper shit eminent domain and Takings analysis. Super shitty like the graders going to see eminent domain think "yeah this dude should not be practicing," and then my takings was jusst shit because I was down to literally seconds at that point. Btw, happy that I at least got through both MBE sections (last time I didnt).

4. Hit search and seizure (govt conduct, S&S, standing? idk forgot already), then went into warrant requirement if all are present, then said warrant was fine after going through several different analyses, then since Officer Eager raced to the house I hit warrantless exceptions, and of them go her in except to Exigent Circumstances??????????????? That's what i called it. Not Hot pursuit, but Exigent Fucking Circumstances. This is literally, if I remember correctly, how I did it too "Warrantless Exception #5 = Exigent Circumstances." Yeah, so fucked that up. But then Eager got in. So next part of the essay was the stuff right, so Bomb is fine bc girl could be hiding in closet, analysis, but other two were not, and did then exclusionary rule analyses for all three. Then, I completely fucked the rule for Kidnapping up and even remember mentioning aggravated kidnapping but no because not for money, but then maybe MAYBE turned it around a bit under Attempted Kidnapping analysis writ large. I thought about writing about how other two could come in if used for a grand jury indictment, but i dont remember if this is correct so I opted not to fuck essay 4 up even more.

5. Wills was a fucking mess. full stop. First, I started by talking about the overall Community Property rule, then right below it I had a little section titled “CA Rule Application” talking about the three areas ways its valid (state, domiciled, death). Next, I talked about validity of 2001 will (present testamentary intent, capacity, and will formalities), but prob did not get the BLL precise for each of those. Then I hit revocation I think (revival, DRR, physical act, subsequent will, codicil). Then codicil, and I think somewhere it the abovementioned analysis I mentioned the CA rule regarding the signature. I think I then went into devises under the will. Start with Nell and hit putative spouse, dismissed, then co-habitants, dismissed, then intestate share, but didn’t get that right, then lapse and anti-lapse, dismissed. Then with Ann I talked about her gift being specific not general because it was for a sum certain from SPECIFICALLY his SP? So definitely fucked that up because it was a general devise… hmmm… that sucks. I also hit Ann on anti-lapse and gave her all that shit from his first will. Bob got his gift and something else, forgot, and Carol got intestate share because he was an omitted child through adoption… CA recognizes adoption as synonymous with blood child, and presumes omission to be unintentional absent evidence to the contrary. No existed here plus intention to give because his other two children were gifted 20k I think. START employers under class gift, can be left open, and probably fucked that rule up too.

PT: this is why I think I will be retaking this again for the fourth time. This is terrible and fucking heartbreaking because this thing has cost me so much over the last almost two years now. It’s also cost others. Anyway. So I had a little over an hour to do the PT and quickly banged through and created my outline: I. Contract Analysis, II. Gift Analysis (a. unconditional, b. conditional), III. Standing Analysis. But then it all went to shit. Didn’t have time. I go all the rules down for each section, but by that time was left with only ten minutes analysis with our facts. So I hit one paragraph on how its not a contract, and maybe four sentences on how it’s an unconditional gift. Finally, I was down to two minutes and literally wrote one sentence dismissing P’s complaint for lack of claim for which relief could be granted given that this was not a contract, so this PT will be a 50, maybe even a 45 or 40.

Out of five essays I think I might have gotten this: 70, 65, 60, 55, 50… maybe those are even less, and a 45 or 50 on the PT.

I am going to get back in the gym, figure out the job situation, sort out my material, get my shit together and next week I will start by reading the Barbri big books I have, like the huge outlines. I am also going to fucking memorize all my subject outlines to the point where I can write shit down from memory. I am doing 15 essays for each subject. Each fucking one. And not just out line them. I am going to do it to where I have these facts patterns memorized and answers via the answers outlined below. I have to think about what I am going to do regarding MBE prep.

This bar is passable because I fucking felt it this time around. But then I felt it slip out on my hands, again, and its just heartbreaking. Some of us are ok and other aren’t. Some of us have lost a little and others have lost a lot. I have lost love. I have lost almost everything. I have nothing right now. But that’s ok because I will push back and keep going, and with the right corrections I will pass. Its just that I am getting sick of this shit. I just turned 35. I have really nothing to show for it. I went into the navy after high school, then undergrad and grad at two great schools, ok, but fuck, this is getting ridiculous. Since the Navy I really have not had much work experience. Highly talented but just dragged down by this shit bar. I should have practice PTs under timed conditions. I should have done essays under timed conditions. I should have known the BLL better. I should have a better and quicker understand substantive law period.

/rant

Thank you for taking the time to read this if you made it this far. Sincerely, I mean that.

Update: I knowing I am freaking and very well might have passed. But then I maybe didn't. I think I passed the MBEs because I honestly felt pretty good both sessions, but if that PT and shit wills essays tank my written portion that will suck but will be a good personal lesson. Gym time.
You’re not alone. I don’t know how many times I told myself “Hit PT first” I read the two essay questions and got sucked in! Ugh. What makes it worse is that the substance of the PT was so freaking easy!! Ugh. Oh well.
Nothing I can do about it so I’m not going to stress it (easier said than done).

Bla Bla Bla Blah

Bronze
Posts: 138
Joined: Thu Mar 01, 2018 1:01 pm

Re: 2018 February CA Bar

Post by Bla Bla Bla Blah » Sun Mar 04, 2018 12:19 am

Perhaps this will help, for anyone who has work, kids, other obligations and needs a quick way to study. I was given a little over 2 and 1/2 weeks to wrap up my arguments against HP and Roku for my firm and prep for this test. Realized that "I ain't got time" pretty fast. Luckily, I have a background in criminal law, and deal with Federal practice (we're talking summary judgement and dismissal issues on a regular basis in patent arguments), and so crim pro, crim law, and quite a bit of evidence and civ pro were confident areas for me (didn't even really touch my material on criminal issues).

Overall, the goal with limited time is to get straight into the meat. By seeing the rule statements so much, I recalled them really well during the bar. Let's face it, on most of these rules (contract formation) we simply need to remember how to state them. We don't need to waste a ton of time having a lecturer tell us what consideration is, or what the mailbox rule is, while we write this into a giant lecture outline (which we are then responsible for breaking down into leansheet format) because we all went to law school. It was way more helpful to see it in simple rule statement form again and again on a flashcard, read the explanation, and speed past the basic rules (focusing more on the tricky ones) while I flicked it into a box. The lectures were helpful only to fill times where I could not actively study with passive learning. I outlined essay subjects (with rule statements, and follow up review of these rule statements, and an idea of how I would analyze) on all CA subjects--and gave special attention to PR.

I honestly would have done exactly what I did with my 2 and 1/2 weeks if I had more time too. Beats wasting time sitting down through slow lectures/all the time spent reinventing the wheel and developing your own outlines with time that should be spent in active learning, MBE testing being the most important to understanding Fed rules and building confidence, in my opinion. Adaptibar almost taught me exactly how to analyze when I studied all their follow up explanations. I also had the advantageof knowing that what I was reading was, at all times, the correct statement of the law and not wasting all that time breaking down my own outlines into 3-6 page leansheet formats--who has time to waste on learning how to break material down when the focus should be on finding the quickest way to digest the rules and giving it back to examiners the way they want ot by testing yourself on them over, and over again?

Felt very good in practice since I was where I needed to be in MBE, and Adaptibar told me how I was being most effective and when. Also knew that I was spotting all the key issues in my essay outlines as my issue spotting was right along with the model answers. I also left no guess work to how I would write and organize my essays--a book on that was helpful. Felt pretty good about it all, but remember, take this with a grain of salt. I still don't know if I passed!

Edit: Forgot to mention this, and honestly I decided on this route during the exam... but tackling the PT first was immensely helpful to my approach. I didn't practice the PT's prior to the exam, but wrote it exactly as I would have in practice. Because I hit the PT first, mental fatigue and the mental stress that comes with being immediately up against the clock did not affect me. I figured that if anything affected my performance, it would be better that this occur on an essay question that was worth 1 point for every 2 points that I could gain on the PT. I probably took an extra 15 to 20 minutes on this question making sure it all looked nice, and doing a quick proof. And while I was about 10-15 minutes short on the wills essay, and faced mental fatigue/time stress on the question, it wasn't enough to affect my approach and I finished with 5 minutes left. If I lost a few points on wills because of the strategy, it definitely beat having a sloppy PT because all of the stress that kicks in when I look at my watch or when the proctor yells "5 minutes left!"

So here's what I did, from most utilized material to least.

Adaptibar ($360): Jumped right in and pushed through as many as I could per day. When you get those wrong, they tend to stick in your head and teach you nuanced issues of the law much better. Always reviewed explanations for each answer, which included full rule statements. Downloaded the app and could test everywhere. Adaptibar even told me my timing for each question, as well as the optimal time for my answers (i.e., at what time I got the most right). This helped me because I knew that if it took me anywhere longer than 1min 15sec to answer, I was probably going to trip myself up and not ger it right--so answer and move on! Obviously because I started on these questions cold, I worked my way up and was hitting 73 percent during my last week (close to 66 percent overall).

Critical pass flashcards ($150): No way in hell I would have had time to create an outline and flashcards through a full course. And I found the Kaplan books my friend sold to me, and commercial outlines were inferior to these cards. I'd grab a topic, and when I was exhausted from everything else I had done, I'd lay there flick through all the cards for the topic (usually right before I was ready to shut it down for the night), flicking them into the lid for the box. Very nice, and full, and well broken down explanations. I would make myself recite the rules outloud, until I could do it from memory, on some of the trickier rules (covenants of title, lemon test, obscenity test, Daubert, etc.). But most importantly, it was a great way to constantly cycle through the subjects.

($45 used) I bought a book, and don't recall the name of it, on how to write for the CA Bar. It had good reviews on Amazon, and gave me a good idea of how to save time in the essays by keeping titles succinct (one to two words if possible), only using bold (too many things to worry about turning on/off when you're capped, Ital, Underline, etc.), and creating a quick outline. Really helpful on the CA subjects, and discussed the most important topics to bring up in each. Incuded questions, model answers, and a breakdown of why.

MP3 Lectures (Free): My friend had MP3 lectures of all the federal subjects. Had those lectures going at all times that I couldn't actively study (in my car, while I was going to sleep, and until I woke up in the morning and got started on active review). Helpful AF!

Leansheets ($30): Not sure that these had the best statements of law, and found myself digging through the box of Kaplan books I purchased for better explanations, but reviewed these over and over again, mostly on CA subjects that were not part of critical pass, was huge in beating CA rules into my head in a way that let me recall them much easier. The book on essays for CA bar were also instructive, by breaking down the areas of CA law to general theme based outlines (which was based on what was most tested on the essays). So I had a theme based overview (helpful to writing) and a full overview because these 3 to 6 page outlines covered everything, and distinguished Fed/ABA from CA very well. It was also nice to have confidence that, even though I didn't write these, they were correct.

Kaplan books ($50): very rarely did I need these, but where a lean sheet rule was not ringing any bells, I'd refer to the outlines in these for clarifications. Again, as with lean sheets, this is pretty much limited to CA law.

Bla Bla Bla Blah

Bronze
Posts: 138
Joined: Thu Mar 01, 2018 1:01 pm

Re: 2018 February CA Bar

Post by Bla Bla Bla Blah » Sun Mar 04, 2018 2:12 am

One more thing. The Critical Pass flashcards had a space where you could write things that helped your understanding of the law. This was on the right hand side, on the back of the card where the explanation was. On Evidence, and Civil Procedure, I wrote the CA differences (which were already stated and understood nicely enough from the commercial Leansheets I had purchased) into the margin, which was helpful in me being able to remember which Fed rules were the same as CA, and which were different. This didn't take to much time. Often, it was as easy as writing "CA: present sense impression must be limited to a statement explaining one's own conduct," or "CA: prior inconsistent statement--no need for under oath for both substantive evidence and impeachment." I'm sure that there were enough blank cards in there that one could make sets of CA subject flashcards too, but the CA essay writing book, and Leansheets seemed to do the trick for any subjects that were not already categorized in the critical pass cards.

They also had an app, and these acted like very general outlines of everything in each Fed subject, but also let me see all the topics really easily and study them on the go (for e.g., if I looked at "content based restrictions" and was at all like "huh?" on the topic, I would tap the subject on my phone and it would show me the flash card explanation--very nice).

sophophone

New
Posts: 2
Joined: Fri Mar 02, 2018 8:28 pm

Re: 2018 February CA Bar

Post by sophophone » Sun Mar 04, 2018 7:34 am

Don't be so hard on yourself! Like you said, you may have passed. I'm with you on the PT. I really messed up on the time. I thought I had 1/2 hour left and then heard the guy say "5 minutes to go." What??? I hadn't even started on the second question! I just wrote really fast that it was a conditional gift and never discussed standing. I read the case in the library about standing, but really didn't see where it fit into the questions. So, you are not alone. I am going to start studying now for July (although I said that after the July 2017 bar and didn't do it). I'm 20 years older than you, so don't worry about your age. We will do this!

RezIpsy wrote:
rayforoc wrote:Long read if you're feeling it. Sincerest apolygizes for all the cussing but I wanted to write how I am actually feeling.

So I am taking it again. TAKING IT AGAIN (Al Pacino from Scent of a Woman Voice).

From what I can recall. This is my wrap up and I will be brutally honest about my prep too.

Prep:
- I did 1100 adaptibar questions ending at 77%;
- went through S&T;
- did all four tests on NBCE website during final week (85%, 90%, 90%, and 88%);
- bought NCBE newest study guide and only got through the first ten civ pro questions;
- never read one outline because I though I remembered shit from July, and realize now how fucking careless and stupid that was.

Essays:
- Outlined (and I mean like almost wrote them out) 10 Torts, 10 Contracts, 8 Evidence + 2 CA Evidence, 10 Property, 9 Con Law, 10 BA + 2 Partnerships and 2 Agency, 5 Trusts, and 6 Wills, 10 Crimes... I didnt touch any civ pro, comm prop, pr, or remedies.

PT:
- did not do any prep here other than to browse through some notes from July.

I didnt outline pr and remedies because I ran out of time and should have prioritized those but didnt because I thought that doing pr and remedies within the BA and Contracts essays I was outlining would be sufficient. This is stupid. Also, not practicing any fucking PTs was stupid, and not fucking timing myself, at all, was stupid (see below to find out why). Like this is the one fucking area where we cannot be arrogant when prepping. I am fucking arrogant sometimes and arrogant bc of so many reasons no of which are important and all of which are stupid, and I think its like actually an infection that is fucking my life up slowly (here because only an idiot would not read outlines, completely skip PT prep, oh and not touch pr and remedies--the two guaranteed subjects... fucking go punish yourself). Arrogance is terrible. Its nasty. It hurts people, and it hurts yourself in my ways than you can imagine. and I am trying to get better each day at this side of me. I Looked through all the comments thus far and you ladies and gents are beasts (you should all be proud), and I am too, its just that I am kind of reckless and I am afraid it really fucked me up this time. The first time I took this I scored something like 1292, and then 1377. I hope I passed but yeah. Hope. The operative word. Hope. A noun. To want something to happen. Use it in a sentence. I unfortunately hope that a certain unqualified and worthless Congresswoman wins a certain midterm election in my district because I would hate to have to run against someone in my own party in 2020 and utterly fuck their lives up in a primary battle.

The Exam

MBEs: Nothing major and felt good both times although I missed a couple easy ones because I forgot the BLL, and the afternoon session was a bit shit, so I am assuming I got one of the easier AM sets seeing as how mostly everyone thought their AM was hard and afternoon easy. Outlines, bud, should have read your fucking outlines. Should have gone through all the questions in the study aid too. Like that's just nonsense to fuck that up. Btw, NCBE needs to write better civ pro questions.

Essays:

1. Hit formation re contract analysis (prob a wait and might actually piss the grader off?), one sentence on terms, then dove into misrepresention and non-disclosure. Then hit PR. From what I rememeber it was Duty of Care, Duty of Loyalty (specifically, conflict of interest and I said lawyer's intersest in saving money with shady expert v. client's interests of winning the case, something like that and worded better), Duty of Dignity and Dicorum, Duty of Competence, Duty to Communicate, and maybe one more? My BLL was absolutely not spot on on some of them (Hence the mistake of not study PR omg).

2. Hit Standing, Organizational Standing (quickly dismissed it tho I think, right? yeah I dont think this worked), then Taxpayer standing (bc dude can get it on this if his/her cause of action is related to one of the religious clauses, right?, then ripeness, then hit constitutional claims and started with the requirement of state action, then went into the religions first, specifically the Establishment Clause first but forget to talk about sect/no sect preference, and did do Lemon here, I think I fucked it up and did Lemon for the Free Exercise Clause, then hit P&I of the 14th but did not hit Due Process, then hit EP and did classification as something like prisoners who are allowed to use/adhere to their recognized religions v. the dude, and since it was religion I thought that was a fundamental right and applied strict scrutiny, also, I distinctly remember doing separate analysis for the book and for the tea on several of the issues.

3. Hit Public Nuisance, then private nuisance exception under public, then hit Private Nuisance, then Trespass (and didnt call it Trespass To Land But Rather just Trespass), then Tresspass To Chattels (wire fence), then hit Negligence cuz 10 years of this shit is fucking negligent (went through quick analysis with both cardozo and andrews standard of care included). Do it matter which way you landed on for all three nuisance claims? I forgot which way I went. I feel like instantly it does matter. Didnt do conversion, IIED, Neg IIED, but did do a super, suuuuper shit eminent domain and Takings analysis. Super shitty like the graders going to see eminent domain think "yeah this dude should not be practicing," and then my takings was jusst shit because I was down to literally seconds at that point. Btw, happy that I at least got through both MBE sections (last time I didnt).

4. Hit search and seizure (govt conduct, S&S, standing? idk forgot already), then went into warrant requirement if all are present, then said warrant was fine after going through several different analyses, then since Officer Eager raced to the house I hit warrantless exceptions, and of them go her in except to Exigent Circumstances??????????????? That's what i called it. Not Hot pursuit, but Exigent Fucking Circumstances. This is literally, if I remember correctly, how I did it too "Warrantless Exception #5 = Exigent Circumstances." Yeah, so fucked that up. But then Eager got in. So next part of the essay was the stuff right, so Bomb is fine bc girl could be hiding in closet, analysis, but other two were not, and did then exclusionary rule analyses for all three. Then, I completely fucked the rule for Kidnapping up and even remember mentioning aggravated kidnapping but no because not for money, but then maybe MAYBE turned it around a bit under Attempted Kidnapping analysis writ large. I thought about writing about how other two could come in if used for a grand jury indictment, but i dont remember if this is correct so I opted not to fuck essay 4 up even more.

5. Wills was a fucking mess. full stop. First, I started by talking about the overall Community Property rule, then right below it I had a little section titled “CA Rule Application” talking about the three areas ways its valid (state, domiciled, death). Next, I talked about validity of 2001 will (present testamentary intent, capacity, and will formalities), but prob did not get the BLL precise for each of those. Then I hit revocation I think (revival, DRR, physical act, subsequent will, codicil). Then codicil, and I think somewhere it the abovementioned analysis I mentioned the CA rule regarding the signature. I think I then went into devises under the will. Start with Nell and hit putative spouse, dismissed, then co-habitants, dismissed, then intestate share, but didn’t get that right, then lapse and anti-lapse, dismissed. Then with Ann I talked about her gift being specific not general because it was for a sum certain from SPECIFICALLY his SP? So definitely fucked that up because it was a general devise… hmmm… that sucks. I also hit Ann on anti-lapse and gave her all that shit from his first will. Bob got his gift and something else, forgot, and Carol got intestate share because he was an omitted child through adoption… CA recognizes adoption as synonymous with blood child, and presumes omission to be unintentional absent evidence to the contrary. No existed here plus intention to give because his other two children were gifted 20k I think. START employers under class gift, can be left open, and probably fucked that rule up too.

PT: this is why I think I will be retaking this again for the fourth time. This is terrible and fucking heartbreaking because this thing has cost me so much over the last almost two years now. It’s also cost others. Anyway. So I had a little over an hour to do the PT and quickly banged through and created my outline: I. Contract Analysis, II. Gift Analysis (a. unconditional, b. conditional), III. Standing Analysis. But then it all went to shit. Didn’t have time. I go all the rules down for each section, but by that time was left with only ten minutes analysis with our facts. So I hit one paragraph on how its not a contract, and maybe four sentences on how it’s an unconditional gift. Finally, I was down to two minutes and literally wrote one sentence dismissing P’s complaint for lack of claim for which relief could be granted given that this was not a contract, so this PT will be a 50, maybe even a 45 or 40.

Out of five essays I think I might have gotten this: 70, 65, 60, 55, 50… maybe those are even less, and a 45 or 50 on the PT.

I am going to get back in the gym, figure out the job situation, sort out my material, get my shit together and next week I will start by reading the Barbri big books I have, like the huge outlines. I am also going to fucking memorize all my subject outlines to the point where I can write shit down from memory. I am doing 15 essays for each subject. Each fucking one. And not just out line them. I am going to do it to where I have these facts patterns memorized and answers via the answers outlined below. I have to think about what I am going to do regarding MBE prep.

This bar is passable because I fucking felt it this time around. But then I felt it slip out on my hands, again, and its just heartbreaking. Some of us are ok and other aren’t. Some of us have lost a little and others have lost a lot. I have lost love. I have lost almost everything. I have nothing right now. But that’s ok because I will push back and keep going, and with the right corrections I will pass. Its just that I am getting sick of this shit. I just turned 35. I have really nothing to show for it. I went into the navy after high school, then undergrad and grad at two great schools, ok, but fuck, this is getting ridiculous. Since the Navy I really have not had much work experience. Highly talented but just dragged down by this shit bar. I should have practice PTs under timed conditions. I should have done essays under timed conditions. I should have known the BLL better. I should have a better and quicker understand substantive law period.

/rant

Thank you for taking the time to read this if you made it this far. Sincerely, I mean that.

Update: I knowing I am freaking and very well might have passed. But then I maybe didn't. I think I passed the MBEs because I honestly felt pretty good both sessions, but if that PT and shit wills essays tank my written portion that will suck but will be a good personal lesson. Gym time.
You’re not alone. I don’t know how many times I told myself “Hit PT first” I read the two essay questions and got sucked in! Ugh. What makes it worse is that the substance of the PT was so freaking easy!! Ugh. Oh well.
Nothing I can do about it so I’m not going to stress it (easier said than done).

Register now!

Resources to assist law school applicants, students & graduates.

It's still FREE!


Mxmasterr

New
Posts: 42
Joined: Thu Mar 01, 2018 10:14 am

Re: 2018 February CA Bar

Post by Mxmasterr » Sun Mar 04, 2018 9:02 am

RezIpsy wrote:
rayforoc wrote:Long read if you're feeling it. Sincerest apolygizes for all the cussing but I wanted to write how I am actually feeling.

So I am taking it again. TAKING IT AGAIN (Al Pacino from Scent of a Woman Voice).

From what I can recall. This is my wrap up and I will be brutally honest about my prep too.

Prep:
- I did 1100 adaptibar questions ending at 77%;
- went through S&T;
- did all four tests on NBCE website during final week (85%, 90%, 90%, and 88%);
- bought NCBE newest study guide and only got through the first ten civ pro questions;
- never read one outline because I though I remembered shit from July, and realize now how fucking careless and stupid that was.

Essays:
- Outlined (and I mean like almost wrote them out) 10 Torts, 10 Contracts, 8 Evidence + 2 CA Evidence, 10 Property, 9 Con Law, 10 BA + 2 Partnerships and 2 Agency, 5 Trusts, and 6 Wills, 10 Crimes... I didnt touch any civ pro, comm prop, pr, or remedies.

PT:
- did not do any prep here other than to browse through some notes from July.

I didnt outline pr and remedies because I ran out of time and should have prioritized those but didnt because I thought that doing pr and remedies within the BA and Contracts essays I was outlining would be sufficient. This is stupid. Also, not practicing any fucking PTs was stupid, and not fucking timing myself, at all, was stupid (see below to find out why). Like this is the one fucking area where we cannot be arrogant when prepping. I am fucking arrogant sometimes and arrogant bc of so many reasons no of which are important and all of which are stupid, and I think its like actually an infection that is fucking my life up slowly (here because only an idiot would not read outlines, completely skip PT prep, oh and not touch pr and remedies--the two guaranteed subjects... fucking go punish yourself). Arrogance is terrible. Its nasty. It hurts people, and it hurts yourself in my ways than you can imagine. and I am trying to get better each day at this side of me. I Looked through all the comments thus far and you ladies and gents are beasts (you should all be proud), and I am too, its just that I am kind of reckless and I am afraid it really fucked me up this time. The first time I took this I scored something like 1292, and then 1377. I hope I passed but yeah. Hope. The operative word. Hope. A noun. To want something to happen. Use it in a sentence. I unfortunately hope that a certain unqualified and worthless Congresswoman wins a certain midterm election in my district because I would hate to have to run against someone in my own party in 2020 and utterly fuck their lives up in a primary battle.

The Exam

MBEs: Nothing major and felt good both times although I missed a couple easy ones because I forgot the BLL, and the afternoon session was a bit shit, so I am assuming I got one of the easier AM sets seeing as how mostly everyone thought their AM was hard and afternoon easy. Outlines, bud, should have read your fucking outlines. Should have gone through all the questions in the study aid too. Like that's just nonsense to fuck that up. Btw, NCBE needs to write better civ pro questions.

Essays:

1. Hit formation re contract analysis (prob a wait and might actually piss the grader off?), one sentence on terms, then dove into misrepresention and non-disclosure. Then hit PR. From what I rememeber it was Duty of Care, Duty of Loyalty (specifically, conflict of interest and I said lawyer's intersest in saving money with shady expert v. client's interests of winning the case, something like that and worded better), Duty of Dignity and Dicorum, Duty of Competence, Duty to Communicate, and maybe one more? My BLL was absolutely not spot on on some of them (Hence the mistake of not study PR omg).

2. Hit Standing, Organizational Standing (quickly dismissed it tho I think, right? yeah I dont think this worked), then Taxpayer standing (bc dude can get it on this if his/her cause of action is related to one of the religious clauses, right?, then ripeness, then hit constitutional claims and started with the requirement of state action, then went into the religions first, specifically the Establishment Clause first but forget to talk about sect/no sect preference, and did do Lemon here, I think I fucked it up and did Lemon for the Free Exercise Clause, then hit P&I of the 14th but did not hit Due Process, then hit EP and did classification as something like prisoners who are allowed to use/adhere to their recognized religions v. the dude, and since it was religion I thought that was a fundamental right and applied strict scrutiny, also, I distinctly remember doing separate analysis for the book and for the tea on several of the issues.

3. Hit Public Nuisance, then private nuisance exception under public, then hit Private Nuisance, then Trespass (and didnt call it Trespass To Land But Rather just Trespass), then Tresspass To Chattels (wire fence), then hit Negligence cuz 10 years of this shit is fucking negligent (went through quick analysis with both cardozo and andrews standard of care included). Do it matter which way you landed on for all three nuisance claims? I forgot which way I went. I feel like instantly it does matter. Didnt do conversion, IIED, Neg IIED, but did do a super, suuuuper shit eminent domain and Takings analysis. Super shitty like the graders going to see eminent domain think "yeah this dude should not be practicing," and then my takings was jusst shit because I was down to literally seconds at that point. Btw, happy that I at least got through both MBE sections (last time I didnt).

4. Hit search and seizure (govt conduct, S&S, standing? idk forgot already), then went into warrant requirement if all are present, then said warrant was fine after going through several different analyses, then since Officer Eager raced to the house I hit warrantless exceptions, and of them go her in except to Exigent Circumstances??????????????? That's what i called it. Not Hot pursuit, but Exigent Fucking Circumstances. This is literally, if I remember correctly, how I did it too "Warrantless Exception #5 = Exigent Circumstances." Yeah, so fucked that up. But then Eager got in. So next part of the essay was the stuff right, so Bomb is fine bc girl could be hiding in closet, analysis, but other two were not, and did then exclusionary rule analyses for all three. Then, I completely fucked the rule for Kidnapping up and even remember mentioning aggravated kidnapping but no because not for money, but then maybe MAYBE turned it around a bit under Attempted Kidnapping analysis writ large. I thought about writing about how other two could come in if used for a grand jury indictment, but i dont remember if this is correct so I opted not to fuck essay 4 up even more.

5. Wills was a fucking mess. full stop. First, I started by talking about the overall Community Property rule, then right below it I had a little section titled “CA Rule Application” talking about the three areas ways its valid (state, domiciled, death). Next, I talked about validity of 2001 will (present testamentary intent, capacity, and will formalities), but prob did not get the BLL precise for each of those. Then I hit revocation I think (revival, DRR, physical act, subsequent will, codicil). Then codicil, and I think somewhere it the abovementioned analysis I mentioned the CA rule regarding the signature. I think I then went into devises under the will. Start with Nell and hit putative spouse, dismissed, then co-habitants, dismissed, then intestate share, but didn’t get that right, then lapse and anti-lapse, dismissed. Then with Ann I talked about her gift being specific not general because it was for a sum certain from SPECIFICALLY his SP? So definitely fucked that up because it was a general devise… hmmm… that sucks. I also hit Ann on anti-lapse and gave her all that shit from his first will. Bob got his gift and something else, forgot, and Carol got intestate share because he was an omitted child through adoption… CA recognizes adoption as synonymous with blood child, and presumes omission to be unintentional absent evidence to the contrary. No existed here plus intention to give because his other two children were gifted 20k I think. START employers under class gift, can be left open, and probably fucked that rule up too.

PT: this is why I think I will be retaking this again for the fourth time. This is terrible and fucking heartbreaking because this thing has cost me so much over the last almost two years now. It’s also cost others. Anyway. So I had a little over an hour to do the PT and quickly banged through and created my outline: I. Contract Analysis, II. Gift Analysis (a. unconditional, b. conditional), III. Standing Analysis. But then it all went to shit. Didn’t have time. I go all the rules down for each section, but by that time was left with only ten minutes analysis with our facts. So I hit one paragraph on how its not a contract, and maybe four sentences on how it’s an unconditional gift. Finally, I was down to two minutes and literally wrote one sentence dismissing P’s complaint for lack of claim for which relief could be granted given that this was not a contract, so this PT will be a 50, maybe even a 45 or 40.

Out of five essays I think I might have gotten this: 70, 65, 60, 55, 50… maybe those are even less, and a 45 or 50 on the PT.

I am going to get back in the gym, figure out the job situation, sort out my material, get my shit together and next week I will start by reading the Barbri big books I have, like the huge outlines. I am also going to fucking memorize all my subject outlines to the point where I can write shit down from memory. I am doing 15 essays for each subject. Each fucking one. And not just out line them. I am going to do it to where I have these facts patterns memorized and answers via the answers outlined below. I have to think about what I am going to do regarding MBE prep.

This bar is passable because I fucking felt it this time around. But then I felt it slip out on my hands, again, and its just heartbreaking. Some of us are ok and other aren’t. Some of us have lost a little and others have lost a lot. I have lost love. I have lost almost everything. I have nothing right now. But that’s ok because I will push back and keep going, and with the right corrections I will pass. Its just that I am getting sick of this shit. I just turned 35. I have really nothing to show for it. I went into the navy after high school, then undergrad and grad at two great schools, ok, but fuck, this is getting ridiculous. Since the Navy I really have not had much work experience. Highly talented but just dragged down by this shit bar. I should have practice PTs under timed conditions. I should have done essays under timed conditions. I should have known the BLL better. I should have a better and quicker understand substantive law period.

/rant

Thank you for taking the time to read this if you made it this far. Sincerely, I mean that.

Update: I knowing I am freaking and very well might have passed. But then I maybe didn't. I think I passed the MBEs because I honestly felt pretty good both sessions, but if that PT and shit wills essays tank my written portion that will suck but will be a good personal lesson. Gym time.
You’re not alone. I don’t know how many times I told myself “Hit PT first” I read the two essay questions and got sucked in! Ugh. What makes it worse is that the substance of the PT was so freaking easy!! Ugh. Oh well.
Nothing I can do about it so I’m not going to stress it (easier said than done).
We got lucky with that pt one page of facts only like 9 of case law mostly just quotes to throw around. I raced through the first two essays to get to the pt. I think next time, if I'm unlucky and have to, I'd just do pt first it's worth soon much, a high pt can make up for two 50s on essays. I didn't like how they said tell us what gift it is and didnt say explicitly bring up standing , but I had time at the end so I said ok here's some standing stuff and also this is likely a charitable trust. Just like a paragraph for each, I was wondering whether I'd get points off for that. The trust test was sooooo on point though that I was like I gotta say this s a charitable trust by law likely

Bliss

New
Posts: 2
Joined: Sun Mar 04, 2018 12:08 pm

Re: 2018 February CA Bar

Post by Bliss » Sun Mar 04, 2018 12:16 pm

Bla Bla Bla Blah wrote:Perhaps this will help, for anyone who has work, kids, other obligations and needs a quick way to study. I was given a little over 2 and 1/2 weeks to wrap up my arguments against HP and Roku for my firm and prep for this test. Realized that "I ain't got time" pretty fast. Luckily, I have a background in criminal law, and deal with Federal practice (we're talking summary judgement and dismissal issues on a regular basis in patent arguments), and so crim pro, crim law, and quite a bit of evidence and civ pro were confident areas for me (didn't even really touch my material on criminal issues).

Overall, the goal with limited time is to get straight into the meat. By seeing the rule statements so much, I recalled them really well during the bar. Let's face it, on most of these rules (contract formation) we simply need to remember how to state them. We don't need to waste a ton of time having a lecturer tell us what consideration is, or what the mailbox rule is, while we write this into a giant lecture outline (which we are then responsible for breaking down into leansheet format) because we all went to law school. It was way more helpful to see it in simple rule statement form again and again on a flashcard, read the explanation, and speed past the basic rules (focusing more on the tricky ones) while I flicked it into a box. The lectures were helpful only to fill times where I could not actively study with passive learning. I outlined essay subjects (with rule statements, and follow up review of these rule statements, and an idea of how I would analyze) on all CA subjects--and gave special attention to PR.

I honestly would have done exactly what I did with my 2 and 1/2 weeks if I had more time too. Beats wasting time sitting down through slow lectures/all the time spent reinventing the wheel and developing your own outlines with time that should be spent in active learning, MBE testing being the most important to understanding Fed rules and building confidence, in my opinion. Adaptibar almost taught me exactly how to analyze when I studied all their follow up explanations. I also had the advantageof knowing that what I was reading was, at all times, the correct statement of the law and not wasting all that time breaking down my own outlines into 3-6 page leansheet formats--who has time to waste on learning how to break material down when the focus should be on finding the quickest way to digest the rules and giving it back to examiners the way they want ot by testing yourself on them over, and over again?

Felt very good in practice since I was where I needed to be in MBE, and Adaptibar told me how I was being most effective and when. Also knew that I was spotting all the key issues in my essay outlines as my issue spotting was right along with the model answers. I also left no guess work to how I would write and organize my essays--a book on that was helpful. Felt pretty good about it all, but remember, take this with a grain of salt. I still don't know if I passed!

Edit: Forgot to mention this, and honestly I decided on this route during the exam... but tackling the PT first was immensely helpful to my approach. I didn't practice the PT's prior to the exam, but wrote it exactly as I would have in practice. Because I hit the PT first, mental fatigue and the mental stress that comes with being immediately up against the clock did not affect me. I figured that if anything affected my performance, it would be better that this occur on an essay question that was worth 1 point for every 2 points that I could gain on the PT. I probably took an extra 15 to 20 minutes on this question making sure it all looked nice, and doing a quick proof. And while I was about 10-15 minutes short on the wills essay, and faced mental fatigue/time stress on the question, it wasn't enough to affect my approach and I finished with 5 minutes left. If I lost a few points on wills because of the strategy, it definitely beat having a sloppy PT because all of the stress that kicks in when I look at my watch or when the proctor yells "5 minutes left!"

So here's what I did, from most utilized material to least.

Adaptibar ($360): Jumped right in and pushed through as many as I could per day. When you get those wrong, they tend to stick in your head and teach you nuanced issues of the law much better. Always reviewed explanations for each answer, which included full rule statements. Downloaded the app and could test everywhere. Adaptibar even told me my timing for each question, as well as the optimal time for my answers (i.e., at what time I got the most right). This helped me because I knew that if it took me anywhere longer than 1min 15sec to answer, I was probably going to trip myself up and not ger it right--so answer and move on! Obviously because I started on these questions cold, I worked my way up and was hitting 73 percent during my last week (close to 66 percent overall).

Critical pass flashcards ($150): No way in hell I would have had time to create an outline and flashcards through a full course. And I found the Kaplan books my friend sold to me, and commercial outlines were inferior to these cards. I'd grab a topic, and when I was exhausted from everything else I had done, I'd lay there flick through all the cards for the topic (usually right before I was ready to shut it down for the night), flicking them into the lid for the box. Very nice, and full, and well broken down explanations. I would make myself recite the rules outloud, until I could do it from memory, on some of the trickier rules (covenants of title, lemon test, obscenity test, Daubert, etc.). But most importantly, it was a great way to constantly cycle through the subjects.

($45 used) I bought a book, and don't recall the name of it, on how to write for the CA Bar. It had good reviews on Amazon, and gave me a good idea of how to save time in the essays by keeping titles succinct (one to two words if possible), only using bold (too many things to worry about turning on/off when you're capped, Ital, Underline, etc.), and creating a quick outline. Really helpful on the CA subjects, and discussed the most important topics to bring up in each. Incuded questions, model answers, and a breakdown of why.

MP3 Lectures (Free): My friend had MP3 lectures of all the federal subjects. Had those lectures going at all times that I couldn't actively study (in my car, while I was going to sleep, and until I woke up in the morning and got started on active review). Helpful AF!

Leansheets ($30): Not sure that these had the best statements of law, and found myself digging through the box of Kaplan books I purchased for better explanations, but reviewed these over and over again, mostly on CA subjects that were not part of critical pass, was huge in beating CA rules into my head in a way that let me recall them much easier. The book on essays for CA bar were also instructive, by breaking down the areas of CA law to general theme based outlines (which was based on what was most tested on the essays). So I had a theme based overview (helpful to writing) and a full overview because these 3 to 6 page outlines covered everything, and distinguished Fed/ABA from CA very well. It was also nice to have confidence that, even though I didn't write these, they were correct.

Kaplan books ($50): very rarely did I need these, but where a lean sheet rule was not ringing any bells, I'd refer to the outlines in these for clarifications. Again, as with lean sheets, this is pretty much limited to CA law.
Would appreciate if you could pass along the name of essay book you used for bar prep-thanks!

Bla Bla Bla Blah

Bronze
Posts: 138
Joined: Thu Mar 01, 2018 1:01 pm

Re: 2018 February CA Bar

Post by Bla Bla Bla Blah » Sun Mar 04, 2018 4:07 pm

Wish I could just screenshot it. It's called Essay Writing for the California Bar Exam.

I read their suggestions, and after reading those, I skipped to the California subjects, read the outlines, practiced on all the questions (at least in outline form-I didn't have the luxury of doing more than one full essay test [to see if my timing was there] because you don't learn as much black letter law by doing analysis... Adaptibar is a much better use of your time with that hour if you know you're good at writing), reviewed the breakdowns, and saw how they were used in the model answers. This meant that I had to recall the important rules for each subject, and then saw how I should be writing the rules and analyzing multiple times through the breakdowns and essay examples. After I ran out of CA subject based essays, I practiced them through the 2017 Bar questions (especially on PR) and model essays, and other past test Qs and As on the CA Bar site.

The book was written in 2012, so the only thing that was off was gay marriage. I liked the way they theme based categorized their subjects, so instead of an outline that would say "ABA rule says, whatever" it would say "Duty of Loyalty" and then list all the items that should fall under this topic. After going through all the questions, breakdown charts, and seeing how they would write essays (and practicing how to set them up myself), I had no questions on how I would tackle everything during the bar. Overall, the goal was to give the graders an easy way to see that you spotted the issues, streamline how you head essay topics (so this wouldn't slow you down), and give me a checklist of topics I would discuss so my mental energy was spent on rule statements and analysis rather than trying to recall what I would talk about next. When I finished up with recision, I knew exactly what to do. I was going to key up the duty of loyalty, and how L might have breached it. If I needed to add anything, I new it would be a subheader under the Duty of Loyalty header, so everything stayed nice and organized.

On bar exam day my essays looked something like this:

2. Lou's Ethical Violations.


Duty of Loyalty.
Here's the rule
Here's the analysis

Duty of Care.
Duty of Competence:
ABA rule. CA distinction.
Here's the analysis.
Expert Testimony Standard:
Daubert, Fed rule. Frye, CA rule.
Analysis on how it would affect competence, and why there weren't enough in the facts to determine either way.

Lawyer Testimony.
ABA rule. CA distinction.
My analysis.

Bla Bla Bla Blah

Bronze
Posts: 138
Joined: Thu Mar 01, 2018 1:01 pm

Re: 2018 February CA Bar

Post by Bla Bla Bla Blah » Sun Mar 04, 2018 4:11 pm

However, my PT was exactly how I would have written it for the partner's in my firm, e.g., the titles clearly indicated what the section analysis would indicate about the gift. So I didn't use the CA essay book for that.

Get unlimited access to all forums and topics

Register now!

I'm pretty sure I told you it's FREE...


jduckits

New
Posts: 17
Joined: Sun Jul 02, 2017 8:40 pm

Re: 2018 February CA Bar

Post by jduckits » Sun Mar 04, 2018 6:26 pm

Bla Bla Bla Blah wrote:Wish I could just screenshot it. It's called Essay Writing for the California Bar Exam.

I read their suggestions, and after reading those, I skipped to the California subjects, read the outlines, practiced on all the questions (at least in outline form-I didn't have the luxury of doing more than one full essay test [to see if my timing was there] because you don't learn as much black letter law by doing analysis... Adaptibar is a much better use of your time with that hour if you know you're good at writing), reviewed the breakdowns, and saw how they were used in the model answers. This meant that I had to recall the important rules for each subject, and then saw how I should be writing the rules and analyzing multiple times through the breakdowns and essay examples. After I ran out of CA subject based essays, I practiced them through the 2017 Bar questions (especially on PR) and model essays, and other past test Qs and As on the CA Bar site.

The book was written in 2012, so the only thing that was off was gay marriage. I liked the way they theme based categorized their subjects, so instead of an outline that would say "ABA rule says, whatever" it would say "Duty of Loyalty" and then list all the items that should fall under this topic. After going through all the questions, breakdown charts, and seeing how they would write essays (and practicing how to set them up myself), I had no questions on how I would tackle everything during the bar. Overall, the goal was to give the graders an easy way to see that you spotted the issues, streamline how you head essay topics (so this wouldn't slow you down), and give me a checklist of topics I would discuss so my mental energy was spent on rule statements and analysis rather than trying to recall what I would talk about next. When I finished up with recision, I knew exactly what to do. I was going to key up the duty of loyalty, and how L might have breached it. If I needed to add anything, I new it would be a subheader under the Duty of Loyalty header, so everything stayed nice and organized.

On bar exam day my essays looked something like this:

2. Lou's Ethical Violations.


Duty of Loyalty.
Here's the rule
Here's the analysis

Duty of Care.
Duty of Competence:
ABA rule. CA distinction.
Here's the analysis.
Expert Testimony Standard:
Daubert, Fed rule. Frye, CA rule.
Analysis on how it would affect competence, and why there weren't enough in the facts to determine either way.

Lawyer Testimony.
ABA rule. CA distinction.
My analysis.
I didn’t bring up loyalty but I discussed Duty to state facts accurately & truthfully.
Attorney testimony - but could remember all 3 sub rules. But did remember CA distinction - Consent...
Duty of Honesty
I also stated the duty to testify truthfully. But I realized later that this one was for a client not a witness. So not sure I’ll get much points here.

Bla Bla Bla Blah

Bronze
Posts: 138
Joined: Thu Mar 01, 2018 1:01 pm

Re: 2018 February CA Bar

Post by Bla Bla Bla Blah » Sun Mar 04, 2018 6:45 pm

Also, sorry for any weird typos. I do almost everything on this site with a phone... my texting is horrid!

Wanted to add that if I pass in May, I'll be selling everything I have. So if you're interested, keep an eye out on this thread. I'll definitely let you all know if I'm done with this exam, and put out an offer on my material. Not sure if we can PM on this (I think I saw a poster say that we can't anymore), but we'll figure it out and It'll be nice to get rid of it all at that point.

legallyblonde21

New
Posts: 3
Joined: Mon Sep 22, 2014 1:22 am

Re: 2018 February CA Bar

Post by legallyblonde21 » Sun Mar 04, 2018 10:32 pm

Freaking out a little about the PT...I specifically did not address standing and only discussed contract/gift/gift type.

I included a sentence about how standing was not being addressed because it was not specifically asked for, but a standing analysis could be done if requested.

I read the prompt at least 5 times and it did not ask for a standing analysis. It mentioned that the partner thought they might have a standing argument, but only asked for a memo about the contact/gift issues.

Was I very wrong not to address standing? I thought about doing it, but actually thought points might be deducted for not following the directions so I didn’t.

Bla Bla Bla Blah

Bronze
Posts: 138
Joined: Thu Mar 01, 2018 1:01 pm

Re: 2018 February CA Bar

Post by Bla Bla Bla Blah » Sun Mar 04, 2018 10:55 pm

legallyblonde21 wrote:Freaking out a little about the PT...I specifically did not address standing and only discussed contract/gift/gift type.

I included a sentence about how standing was not being addressed because it was not specifically asked for, but a standing analysis could be done if requested.

I read the prompt at least 5 times and it did not ask for a standing analysis. It mentioned that the partner thought they might have a standing argument, but only asked for a memo about the contact/gift issues.

Was I very wrong not to address standing? I thought about doing it, but actually thought points might be deducted for not following the directions so I didn’t.
I don't think so. The only clear instruction was to let the requesting partner know if there was a transfer by contract, or whether it was a gift, and if a gift, what kind of gift (i.e., conditional or unconditional). There was a mention of whether Plaintiff even had standing in the sentence prior to this request, but the analysis asked for was only to help her understand whether standing was there through an analysis of contract/gift... which almost sounded like the partner would make her own decision on standing apart from the memo.

However, I did include a section on standing because there was a case devoted to the issue. Also because in practice standing would have been the most important issue--so it was my first section. If you can dismiss the case on lack of standing, then that is your best argument because the judge doesn't even have to reach the merits of the case. It's done.

In short: you did not do anything that indicated you weren't following instructions. You weren't very wrong to not address standing. The prompt, in my opinion, indicated that the partner who assigned the memo would make that decision based on contract/gift analysis alone.

Communicate now with those who not only know what a legal education is, but can offer you worthy advice and commentary as you complete the three most educational, yet challenging years of your law related post graduate life.

Register now, it's still FREE!


jduckits

New
Posts: 17
Joined: Sun Jul 02, 2017 8:40 pm

Re: 2018 February CA Bar

Post by jduckits » Mon Mar 05, 2018 12:05 am

Has anyone ever got higher than a 55 on a pt without finishing it? I didn’t put a conclusion & I have a partial paragraph on standing st the end. However I have headings with law, comparisons, analysis, & sub conclusions for Contracts, Consideration, Gifts, Absolute vs Non-Absolute Gifts, & Standing

Mxmasterr

New
Posts: 42
Joined: Thu Mar 01, 2018 10:14 am

Re: 2018 February CA Bar

Post by Mxmasterr » Mon Mar 05, 2018 7:36 am

Bla Bla Bla Blah wrote:
legallyblonde21 wrote:Freaking out a little about the PT...I specifically did not address standing and only discussed contract/gift/gift type.

I included a sentence about how standing was not being addressed because it was not specifically asked for, but a standing analysis could be done if requested.

I read the prompt at least 5 times and it did not ask for a standing analysis. It mentioned that the partner thought they might have a standing argument, but only asked for a memo about the contact/gift issues.

Was I very wrong not to address standing? I thought about doing it, but actually thought points might be deducted for not following the directions so I didn’t.
I don't think so. The only clear instruction was to let the requesting partner know if there was a transfer by contract, or whether it was a gift, and if a gift, what kind of gift (i.e., conditional or unconditional). There was a mention of whether Plaintiff even had standing in the sentence prior to this request, but the analysis asked for was only to help her understand whether standing was there through an analysis of contract/gift... which almost sounded like the partner would make her own decision on standing apart from the memo.

However, I did include a section on standing because there was a case devoted to the issue. Also because in practice standing would have been the most important issue--so it was my first section. If you can dismiss the case on lack of standing, then that is your best argument because the judge doesn't even have to reach the merits of the case. It's done.

In short: you did not do anything that indicated you weren't following instructions. You weren't very wrong to not address standing. The prompt, in my opinion, indicated that the partner who assigned the memo would make that decision based on contract/gift analysis alone.
It was a weird prompt. I had time so i reread the thing a few times. It definitely said they wanted us to focus on gift v contract and then tell them what type of gift it was. However it never said do not talk about standing, it did that with the fact section, it said the other lawyer thought she could figure out standing with that analysis.
I figured well they gave it to us, and I suspect the lawyer would want to know so I kinda put it out in a paragraph where I was like a this may help with your standing argument .

I threw out it could be a conditional gift or a per session charitable trust. That one case said anytime u give to an educational institutions for an educational purposes it's a charitable trust by law. It just seemed directly on point, are facts were exactly that it be a per session charitable trust ... And a trust at least in the real world is type of gift device.

hope2018

New
Posts: 29
Joined: Thu Mar 01, 2018 5:57 pm

Re: 2018 February CA Bar

Post by hope2018 » Mon Mar 05, 2018 2:11 pm

Mxmasterr wrote:
Bla Bla Bla Blah wrote:
legallyblonde21 wrote:Freaking out a little about the PT...I specifically did not address standing and only discussed contract/gift/gift type.

I included a sentence about how standing was not being addressed because it was not specifically asked for, but a standing analysis could be done if requested.

I read the prompt at least 5 times and it did not ask for a standing analysis. It mentioned that the partner thought they might have a standing argument, but only asked for a memo about the contact/gift issues.

Was I very wrong not to address standing? I thought about doing it, but actually thought points might be deducted for not following the directions so I didn’t.
I don't think so. The only clear instruction was to let the requesting partner know if there was a transfer by contract, or whether it was a gift, and if a gift, what kind of gift (i.e., conditional or unconditional). There was a mention of whether Plaintiff even had standing in the sentence prior to this request, but the analysis asked for was only to help her understand whether standing was there through an analysis of contract/gift... which almost sounded like the partner would make her own decision on standing apart from the memo.

However, I did include a section on standing because there was a case devoted to the issue. Also because in practice standing would have been the most important issue--so it was my first section. If you can dismiss the case on lack of standing, then that is your best argument because the judge doesn't even have to reach the merits of the case. It's done.

In short: you did not do anything that indicated you weren't following instructions. You weren't very wrong to not address standing. The prompt, in my opinion, indicated that the partner who assigned the memo would make that decision based on contract/gift analysis alone.
It was a weird prompt. I had time so i reread the thing a few times. It definitely said they wanted us to focus on gift v contract and then tell them what type of gift it was. However it never said do not talk about standing, it did that with the fact section, it said the other lawyer thought she could figure out standing with that analysis.
I figured well they gave it to us, and I suspect the lawyer would want to know so I kinda put it out in a paragraph where I was like a this may help with your standing argument .

I threw out it could be a conditional gift or a per session charitable trust. That one case said anytime u give to an educational institutions for an educational purposes it's a charitable trust by law. It just seemed directly on point, are facts were exactly that it be a per session charitable trust ... And a trust at least in the real world is type of gift device.


ugh...I forgot to mention charitable trust under conditional, I said it is non commercial transaction for purpose of supporting education...did not include charitable trust.

Mxmasterr

New
Posts: 42
Joined: Thu Mar 01, 2018 10:14 am

Re: 2018 February CA Bar

Post by Mxmasterr » Mon Mar 05, 2018 2:53 pm

hope2018 wrote:
Mxmasterr wrote:
Bla Bla Bla Blah wrote:
legallyblonde21 wrote:Freaking out a little about the PT...I specifically did not address standing and only discussed contract/gift/gift type.

I included a sentence about how standing was not being addressed because it was not specifically asked for, but a standing analysis could be done if requested.

I read the prompt at least 5 times and it did not ask for a standing analysis. It mentioned that the partner thought they might have a standing argument, but only asked for a memo about the contact/gift issues.

Was I very wrong not to address standing? I thought about doing it, but actually thought points might be deducted for not following the directions so I didn’t.
I don't think so. The only clear instruction was to let the requesting partner know if there was a transfer by contract, or whether it was a gift, and if a gift, what kind of gift (i.e., conditional or unconditional). There was a mention of whether Plaintiff even had standing in the sentence prior to this request, but the analysis asked for was only to help her understand whether standing was there through an analysis of contract/gift... which almost sounded like the partner would make her own decision on standing apart from the memo.

However, I did include a section on standing because there was a case devoted to the issue. Also because in practice standing would have been the most important issue--so it was my first section. If you can dismiss the case on lack of standing, then that is your best argument because the judge doesn't even have to reach the merits of the case. It's done.

In short: you did not do anything that indicated you weren't following instructions. You weren't very wrong to not address standing. The prompt, in my opinion, indicated that the partner who assigned the memo would make that decision based on contract/gift analysis alone.
It was a weird prompt. I had time so i reread the thing a few times. It definitely said they wanted us to focus on gift v contract and then tell them what type of gift it was. However it never said do not talk about standing, it did that with the fact section, it said the other lawyer thought she could figure out standing with that analysis.
I figured well they gave it to us, and I suspect the lawyer would want to know so I kinda put it out in a paragraph where I was like a this may help with your standing argument .

I threw out it could be a conditional gift or a per session charitable trust. That one case said anytime u give to an educational institutions for an educational purposes it's a charitable trust by law. It just seemed directly on point, are facts were exactly that it be a per session charitable trust ... And a trust at least in the real world is type of gift device.


ugh...I forgot to mention charitable trust under conditional, I said it is non commercial transaction for purpose of supporting education...did not include charitable trust.
I struggled with whether to include anything about standing or the trust tbh cause of the way the question was written. But i was like well the there is no other reason for this third case and our facts match up perfectly with a charitable trust by law rule, so i just did maybe like 4 sentences max on standing and that this is probably considered a charitable trust.

Seriously? What are you waiting for?

Now there's a charge.
Just kidding ... it's still FREE!


Post Reply Post Anonymous Reply  

Return to “Bar Exam Prep and Discussion Forum”